The Art and Craft of Problem Solving

(Ann) #1

338 CHAPTER 9 CALCULUS


Symmetry and Transformations


Problem 3.1 .26 on page 73 asked for the evaluation of the preposterously nasty integral

r/^2 dx


Jo 1 + (tanx)V2·

We assume you have already solved this--on your own after studying the similar-but­
easier Example 3.1.7 or with the assistance of the hint online. If not, here is a brief
sketch of a solution.
Obviously, the v'2 exponent is a red herring, so we make it an arbitrary value a
and consider
dx
f(x) :

=
1 + (tanx)a
The values of f(x) range from 1 to 0 as x ranges from 0 to n/2, with f(n/4) = 1/2
right smack in the middle. This suggests that the graph y = f(x) is "symmetric" with
respect to the central point (n/4,1/2), which in turn suggests that we look at the
transformed image of f(x), namely

g(x) := f(n/2 -x).


Armed with trig knowledge [for example, tan(n/ 2 - x) = cotx = l/tanx] the problem
resolves quickly, for it is easy to check that f(x) + g(x) = 1 for all x. Since

rn/2 rn/2
J

o f(x)dx =
J

o g(x)dx,

we're done; the integral equals n / 4.
Why beat this easy problem to death? To remind you that symmetry comes in
many forms. For example, we can say that two points are symmetric with respect to
circle inversion (introduced on page 307). This is just as valid a symmetry as a mere
reflection or rotation. So let's extract the most from the solution to Problem 3.1 .26.
It worked because there was a transformation x � n /2 - x that was invariant with
respect to integration, and that allowed us to simplify the integrand in the problem.
The moral of the story: search for "symmetry," by looking for "natural" transfor­
mations and invariants. Here's a challenging example.
Example 9.3.10 (Putnam 1993) Show that

l-lO( x 2 _X )^2 jir( X2_x )^2 rhl( x (^2) -x )^2


-^100 x^3 - 3x+^1


dx+
Ibl x^3 - 3x+ 1

dx +
Jf&\ x^3 - 3x + 1

dx

is a rational number.

Partial Solution: We sketch the idea, leaving the details to you. This is a very
contrived problem; the limits of integration are not at all random. Call the square root
of the integrand (same for all three terms) f(x) = (x^2 - x)/(x^3 - 3x + 1). Can you find
a transformation that either leaves f(x)^2 alone, or changes it in an instructive way, and
that does something sensible to the limits of integration?
Free download pdf